\documenttype{article} % This is a GELLMU file used to generate the HTML and LaTeX versions. % \base{href="file://localhost/math/local/dept/prelims/hammond/"} \surtitle{} \title{Ph.D. Preliminary Examination in Algebra} \date{January 25, 1999} \nogratlinks \nobanner \latexcommand{\bsl;setlength\{\bsl;topmargin\}\{-36bp\}} \latexcommand{\bsl;setlength\{\bsl;textheight\}\{720bp\}} \begin{document} \begin{enumerate} \item Let $G$ be a finite group and $\mbox{Perm}(G)$ the group of permutations of $G$ viewed as a set. \begin{enumerate} \item Show that the map \[ \lambda;: \spc G \hsp\longrightarrow\hsp \mbox{Perm}(G) \] that is defined by \hsp$\lambda(\sigma)(\tau) = \sigma \circ \tau$\hsp is a group homomorphism. \item Show that the map $\rho;_1$ defined by \hsp$\rho;_1(\sigma)(\tau) = \tau \circ \sigma$\hsp is a homomorphism if and only if $G$ is an abelian group. \item Show that the map $\rho$ defined by \hsp$\rho(\sigma)(\tau) = \tau \circ \sigma^{-1}$\hsp is a homomorphism for every group $G$. \end{enumerate} \item Let $A$ be an $n \times n$ matrix in a field $K$, \hsp let $c(t)$ be the \emph{characteristic polynomial} of $A$, \hsp and let $m(t)$ be the \emph{minimal polynomial} of $A$. Show that $m(t)$ divides $c(t)$ in the polynomial ring $K[t]$. \item Show that the alternating group $A_4$ has no subgroup of index $2$. \item Let \hsp;$f(x) = x^5 - 2$\hsp; in $\mbox{\bold{Q}}\lsb;x\rsb;$, and let $K$ be the splitting field of $f(x)$ over \bold{Q}. \begin{enumerate} \item Find generators for $K$ as a $\mbox{\bold{Q}}$-algebra. \item Find the Galois group $G$ of $K$ over \bold{Q}. \item For each subgroup $H$ of $G$ describe the subfield of $K$ which corresponds to $H$ under the \quophrase{fundamental correspondence of Galois theory}. \end{enumerate} \item Show that if a finite ring $R$ admits an injective (ring) homomorphism from a field,\hsp; then the number of elements of $R$ must be a power of a prime number. \item Let $R$ be a commutative ring, $H$ a commutative $R$-algebra, \hsp and $I$ an ideal in $H$. Show that \[ H/I \hsp\otimes_R\hsp H/I \spc\cong\spc \frac{H \hsp\otimes_R\hsp H}{ I \hsp\otimes_R\hsp H \spc+\spc H \hsp\otimes_R\hsp I} \spc\spc \eos \] \item Let the field $L$ be a (finite) Galois extension of the field $K$. Define \hsp;$\mbox{tr}:\hsp L \hsp\rightarrow\hsp K$\hsp; by \[ \mbox{tr}(\alpha) \spc = \spc \sum_{\sigma\in G} \sigma(\alpha)\sum: \eos \] Show that this \emph{trace} map is surjective on $K$. \item Let $R$ be a ring and $P$ a left $R$-module. Show that the following two statements are equivalent: \begin{enumerate} \item $P$ is a direct summand of a finitely-generated free left $R$-module. \item There exist \hsp;$x_1, \ldots, x_n \in P$, and \hsp;$f_1, \ldots, f_n \in \mbox{Hom}_{R}(P, R)$\hsp; such that the relation \[ x \spc = \spc \sum f_i(x) x_i \sum: \] holds for all $x \in P$. \end{enumerate} \end{enumerate} \end{document}